Monster among the algebra

Find number of triples of integers ( x , y , z ) (x, y ,z) satisfying

x 6 + x 3 + x 3 y + y = 147 157 x 3 + x 3 y + y 2 + y + z 9 = 157 147 \large\ {{ x }^{ 6 } + { x }^{ 3 } + { x }^{ 3 }y + y = { 147 }^{ 157 } \\ { x }^{ 3 } +{ x }^{ 3 }y + { y }^{ 2 } + y + { z }^{ 9 } = { 157 }^{ 147 }}


The answer is 0.

This section requires Javascript.
You are seeing this because something didn't load right. We suggest you, (a) try refreshing the page, (b) enabling javascript if it is disabled on your browser and, finally, (c) loading the non-javascript version of this page . We're sorry about the hassle.

1 solution

Brian Reinhart
May 15, 2018

This problem is USAMO 2005 Problem 2 . This solution is my own creation; it is probably most similar to Solution 2 on the AoPS page.

First and foremost, note that the equations can be rewritten as

( x 3 + 1 ) ( x 3 + y ) = 14 7 157 ( y + 1 ) ( x 3 + y ) + z 9 = 15 7 147 (x^3+1)(x^3+y) = 147^{157} \\ (y+1)(x^3+y) + z^9 = 157^{147}

Since x 3 + 1 14 7 157 x^3+1 | 147^{157} , x x must be even (This will become useful much later). In fact, since 147 = 3 7 2 147 = 3*7^2 , we know that the only primes that can divide x 3 + 1 x^3+1 are 3 and 7. x 3 + 1 x^3+1 factors further into ( x + 1 ) ( x 2 x + 1 ) (x+1)(x^2-x+1) , and gcd ( x + 1 , x 2 x + 1 ) = gcd ( x + 1 , 2 x + 1 ) = gcd ( x + 1 , 3 ) \gcd(x+1,x^2-x+1) = \gcd(x+1,-2x+1) = \gcd(x+1,3) . In particular, only one of x + 1 x+1 and x 2 x + 1 x^2-x+1 can be divisible by 7; more specifically, one of x + 1 x+1 and x 2 x + 1 x^2-x+1 must be a power of 3. (or possibly a negative power of 3).

From here, we split into cases:

  • Case 1: x = 0 x=0 or x = 2 x=2 .

Taking both equations ( m o d 19 ) \pmod{19} gives us, after some consideration,

( x 3 + 1 ) ( x 3 + y ) 2 ( m o d 19 ) ( y + 1 ) ( x 3 + y ) + z 9 11 ( m o d 19 ) (x^3+1)(x^3+y) \equiv 2 \pmod{19} \\ (y+1)(x^3+y)+z^9 \equiv 11 \pmod{19}

If x = 0 x = 0 , then the first equation tells us that y 2 ( m o d 19 ) y \equiv 2 \pmod{19} , and 6 + z 9 11 ( m o d 19 ) z 9 5 ( m o d 19 ) 6+z^9 \equiv 11 \pmod{19} \implies z^9 \equiv 5 \pmod{19} . Since only 1 and -1 are 9th powers ( m o d 19 ) \pmod{19} , this is impossible. Similarly, if x = 2 x=2 , the first equation tells us that 72 + 9 y 2 ( m o d 19 ) 72+9y \equiv 2 \pmod{19} , which can be solved to produce y 7 ( m o d 19 ) y \equiv 7 \pmod{19} . Plugging this into the second equation tells us that 120 + z 9 6 + z 9 11 ( m o d 19 ) 120+z^9 \equiv 6+z^9 \equiv 11 \pmod{19} , which is again impossible.

  • Case 2: x 2 x + 1 x^2-x+1 is a power of 3. [Note that x 2 x x^2-x is always nonnegative for integer values of x x , so we do not need to deal with the possibility that it is negative]

If x 2 x + 1 = 1 x^2-x+1 = 1 , then x 2 x = 0 x^2-x=0 ; since x x is even, we know that x = 0 x=0 , which was covered in case 1. Similarly, if x 2 x + 1 = 3 x^2-x+1=3 , then x 2 x = 2 x^2-x=2 , which gives x = 2 x=2 , which was covered in case 1.

If x 2 x + 1 = 3 k x^2-x+1 = 3^k , with k 2 k \ge 2 , then ( x + 1 ) ( x 2 ) + 3 = 3 k ( x + 1 ) ( x 2 ) = 3 k 3 (x+1)(x-2)+3=3^k \implies (x+1)(x-2)=3^k-3 . But x + 1 x+1 and x 2 x-2 are equivalent ( m o d 3 ) \pmod{3} , so either both are divisible by 3, or neither is. In the first case, 3 2 3 k 3 3^2 | 3^k-3 , which is clearly impossible; in the other case, 3 3 k 3 3 \nmid 3^k-3 , which is also impossible. Thus this case is impossible.

  • Case 3: x + 1 x+1 is a positive power of 3.

Once again, if x + 1 = 1 x+1=1 , x = 0 x=0 , and if x + 1 = 3 x+1=3 , x = 2 x=2 . Both of these cases are dealt with in case 1. In addition, if x + 1 = 9 x+1=9 , x = 8 x=8 , and 8 3 + 1 = 513 8^3+1=513 , which is divisible by 19.

If x + 1 = 3 k x+1=3^k with k 3 k \ge 3 , then we know from the gcd argument at the start that x 2 x + 1 = 3 7 j x^2-x+1=3*7^j , for some positive integer j j . In addition, taking this ( m o d 3 k ) \pmod{3^k} gives us 3 3 7 j ( m o d 3 k ) 1 7 j ( m o d 3 k 1 ) 3 \equiv 3*7^j \pmod{3^k} \implies 1 \equiv 7^j \pmod{3^{k-1}} . By some number theory argument (LTE on 7 j 1 7^j-1 is what I used, but I believe there are other arguments), this implies further that 3 k 2 j 3^{k-2} | j . In particular, since x 2 x + 1 > x 2 > 3 x^2-x+1 > x^2 > 3 , we know that j 3 k 2 j \ge 3^{k-2} . But since x 2 x + 1 < x 2 + 2 x + 1 = 3 2 k x^2-x+1 < x^2+2x+1 = 3^2k , we also have 7 j < 3 2 k 1 7^j < 3^{2k-1} . When k = 3 k = 3 , this gives 7 3 1 = 343 < 3 5 = 243 7^{3^1} = 343 < 3^5 = 243 , which is clearly false. When k > 3 k > 3 , a simple induction argument shows that 3 k 2 > 2 k 1 3^{k-2} > 2k-1 , so since 7 > 3 7>3 , clearly the inequality cannot hold. Thus there are no solutions in this case.

  • Case 4: x + 1 x+1 is a negative power of 3. (that is, the negative of some power of 3)

Here, we have slightly different base cases from case 3: x + 1 = 1 x = 2 x+1=-1 \implies x=-2 , and x + 1 = 3 x = 4 x+1=-3 \implies x=-4 . In these base cases, we can do a similar analysis to that in case 1: x = 2 y = 5 z 9 = 10 x=-2 \implies y=5 \implies z^9=10 , which is impossible, and x = 4 y = 13 z 9 = 3 x=-4 \implies y=13 \implies z^9=3 , which is impossible. Just like in case 3, x + 1 = 9 x = 10 x 3 + 1 = 999 x+1=-9 \implies x=-10 \implies x^3+1=-999 , which is divisible by a number distinct from 3 or 7 (37 in particular). When k 3 k \ge 3 , we once again have that x 2 x + 1 = 3 7 j 7 j 1 ( m o d 3 k 1 ) x^2-x+1=3*7^j \implies 7^j \equiv 1 \pmod{3^{k-1}} , which implies that 3 k 2 j 3^{k-2} | j , and once again j > 0 j > 0 by bounding x 2 x + 1 x^2-x+1 from below. Here's where things get different from case 3: since x x is negative, x 2 x + 1 > x 2 + 2 x + 1 x^2-x+1 > x^2+2x+1 , so instead we have to use x 2 x + 1 < x 2 2 x + 1 = ( x 1 ) 2 = ( 3 k 2 ) 2 = ( 3 k + 2 ) 2 x^2-x+1 < x^2-2x+1 = (x-1)^2 = (-3^k-2)^2 = (3^k+2)^2 . However, after this, the flow is very similar: when k = 3 k = 3 , we need 3 7 3 = 1029 < ( 3 3 + 2 ) 2 = 2 9 2 < 1000 3*7^3=1029 < (3^3+2)^2=29^2 < 1000 , which is clearly impossible. When k > 3 k > 3 , just note that 3 k + 2 < 3 k + 1 3^k+2 < 3^{k+1} , so the inequality becomes 7 j < 3 2 k + 1 7^j < 3^{2k+1} . By another simple induction argument, 3 k 2 2 k + 1 3^{k-2} \ge 2k+1 for all k 4 k \ge 4 , and since 7 > 3 7 > 3 , once again the inequality cannot hold. Thus there are no solutions in this case.

Overall, there are no solutions in any case, so there are 0 \boxed{0} solutions.

Note: in my original solution, I made a mistake and thought ( m o d 19 ) \pmod{19} arguments did not work, so I used much messier ( m o d 73 ) \pmod{73} arguments. For simplicity, I have used the simpler ( m o d 19 ) \pmod{19} arguments in this solution.

0 pending reports

×

Problem Loading...

Note Loading...

Set Loading...